LSAT and Law School Admissions Forum

Get expert LSAT preparation and law school admissions advice from PowerScore Test Preparation.

 Adam Tyson
PowerScore Staff
  • PowerScore Staff
  • Posts: 5153
  • Joined: Apr 14, 2011
|
#46927
There are a few problems with answer E, elewis10. Let's take a look at them one at a time:

1. As our original explanation points out in this thread, answer E moves from possibility (events CAN affect) to certainty (WILL lead). The stimulus never made that leap, but rather started from a conditional claim (WHEN abundant rain falls, hurricanes hit, or ARF :arrow: H). That's not about possibility, but certainty. That mismatch is enough to eliminate answer E. But wait, there's more!

2. The stimulus, as noted above, gave us a conditional premise about the relationship between rain and hurricanes. Answer E instead gave us a CAUSAL premise. That, too, kills this answer.

3. Answer E tells us that events in one place can affect another place, but it doesn't tell us what that effect is. The conclusion is that a certain type of event will cause the SAME type of event in the other place. The author made a leap from AN effect to THE SAME effect! That also did not happen in the stimulus, where the author spelled out for us exactly what the alleged cause and effect were. Answer E dies a thousand deaths!

Note that the stimulus actually had two premises, one causal and one conditional. The flaw was really in the relationship between the conditional premise (or the correlation, if you prefer) and the conclusion, with the causal premise providing a little extra info. None of the answers had two premises, so none of them are a perfect match, but we don't need to find a perfect answer, just the best match for the flaw we were given. Look for a correlation being used to prove a causal relationship and you'll have your best answer.

Get the most out of your LSAT Prep Plus subscription.

Analyze and track your performance with our Testing and Analytics Package.